1
$\begingroup$

Let

$$Y_t:=1+\int_0^t b(s)ds + W_t,\quad\forall t\ge 0,$$

where $b:\mathbb R_+\to[1,2]$ is continuous and $(W_t)_{t\ge 0}$ is a standard Brownian motion. Denote $\tau:=\{t\ge 0: Y_t\le 0\}$ and $X_t:=Y_{t\wedge \tau}$. It is known from On the marginal distributions of an absorbed diffusion that the law of $X_t$, denoted by $\mu_t$, has the following decomposition:

$$\mu_t(dx) = \alpha(t)\delta_0(dx) + p_t(x)dx,\quad \forall t>0.$$

Can we show (under suitable conditions) $p_t(0+):=\lim_{x\to 0+}p_t(x)=0$ for every $t>0$?

PS : When $b$ is constant, e.g. $b\equiv 1$, we have

$$\int_x^{\infty}p_t(y)dy = \mathbb P[X_t>x] = \mathbb P[\inf_{0\le s\le t}Y_s>0, Y_t>x]= \mathbb P[\sup_{0\le s\le t}(-s+W_s)<1, -t+W_t<1-x],\quad \forall t,x>0.$$

Using the joint density of the drifted Brownian Motion and its running maximum, one has

$$\mathbb P[\sup_{0\le s\le t}(-s+W_s)<1, -t+B_t<1-x]=\int_0^1 dm \int_{-\infty}^{1-x}{\bf 1}_{\{y\le m\}} e^{-t/2-y}\frac{2(2m-y)}{\sqrt{2\pi t^3}}e^{-(2m-y)^2/2t}dy,$$

which yields by differentiating with respect to $x$

$$p_t(0+)=-\lim_{x\to 0+} \frac{\partial \mathbb P[X_t>x]}{\partial x}=0.$$

Can we extend to the general function $b$? The key is to show the existence of the joint density of $(Y_t, \inf_{0\le s\le t}Y_s)$ but I do not know how prove it.

$\endgroup$
1
  • $\begingroup$ Could you please provide the reference for the joint density of $(\sup_{0\le s\le t}(-s+W_s), t+W_t)$? My feeling is that the related arguments might be adapted to your case, while I am unable to find its derivation... $\endgroup$
    – user420828
    Dec 1, 2021 at 20:39

1 Answer 1

3
$\begingroup$

We can re-write the problem in terms of $W(t)$ alone, or, even better, in terms of the drifted Brownian motion $\tilde W(t) = W(t) - M t$, where $M$ is the supremum of $|b(s)|$.

Define $$ B(t) = -1 - \int_0^t b(s) ds - M t ,$$ so that $X(t) = \tilde W(t) - B(t)$ up to time $$ \tau = \inf \{ t > 0 : \tilde W(t) \leqslant B(t) \} . $$ Fix $t_0 > 0$ and define $$ \sigma = \inf \{ t \in (0, t_0] : \tilde W(t) \leqslant B(t_0) \} . $$ Since $B$ is a non-increasing function, we clearly have $\sigma \geqslant \tau$, and hence the measure $$\mu(dx) = \mathbb P(t_0 < \tau, \tilde W(t_0) - B(t_0) \in dx)$$ is dominated by the measure $$\nu(dx) = \mathbb P(t_0 < \sigma, \tilde W(t_0) - B(t_0) \in dx) .$$ The latter is, however, just the distribution at time $t_0$ of the drifted Brownian motion $\tilde W(t) - B(t_0)$, killed upon hitting $0$. As you write in the statement of the problem, this is known to have a density function continuously vanishing at zero, and hence $\mu(dx)$ also has a density function continuously vanishing at zero.

It remains to note that $\mu$ is precisely the distribution of $X(t_0)$, up to an extra atom at $0$.


Remark: A more general approach to the problem, which seems to work also when $b(s)$ is an (adapted) stochastic process rather than a deterministic function, would involve showing first that the distribution of $Y(t)$ — and thus also that of $X(t)$ — has a bounded density function (save for an atom at $0$), and then using Chapman–Kolmogorov equation and a comparison argument similar to the one given above to conclude that the density function of the distribution of $X(t)$ goes to zero at $0$.

$\endgroup$
7
  • $\begingroup$ If you mean $W_t$ vs. $W(t)$ — done. (I am so much used to mixing both notations that I do not even notice the difference, sorry.) $\endgroup$ Dec 2, 2021 at 11:35
  • $\begingroup$ I mean $\tilde W_t$ (or $\tilde W(t)$). $W(t)$ is fine as it is consistent here. Thanks so much! $\endgroup$
    – GJC20
    Dec 2, 2021 at 11:37
  • $\begingroup$ Really nice idea! $\endgroup$
    – GJC20
    Dec 2, 2021 at 11:37
  • $\begingroup$ Dear Mateusz, I returned to the case where $b$ is an adapted process, i.e. $b=g(t,Y_t)$ for some suitable function $g$. Could you please detail how to show the density function continuously vanishing at zero? Thank you very much $\endgroup$
    – GJC20
    Apr 4, 2022 at 15:04
  • $\begingroup$ If needed, I can formulate my question in another post $\endgroup$
    – GJC20
    Apr 4, 2022 at 15:05

Your Answer

By clicking “Post Your Answer”, you agree to our terms of service and acknowledge you have read our privacy policy.

Not the answer you're looking for? Browse other questions tagged or ask your own question.